Difference between revisions of "1992 AJHSME Problems/Problem 2"

(Created page with '==Problem== Which of the following is not equal to <math>\dfrac{5}{4}</math>? <math>\text{(A)}\ \dfrac{10}{8} \qquad \text{(B)}\ 1\dfrac{1}{4} \qquad \text{(C)}\ 1\dfrac{3}{12}…')
 
Line 17: Line 17:
 
{{AJHSME box|year=1992|num-b=1|num-a=3}}
 
{{AJHSME box|year=1992|num-b=1|num-a=3}}
 
[[Category:Introductory Algebra Problems]]
 
[[Category:Introductory Algebra Problems]]
 +
{{MAA Notice}}

Revision as of 00:08, 5 July 2013

Problem

Which of the following is not equal to $\dfrac{5}{4}$?

$\text{(A)}\ \dfrac{10}{8} \qquad \text{(B)}\ 1\dfrac{1}{4} \qquad \text{(C)}\ 1\dfrac{3}{12} \qquad \text{(D)}\ 1\dfrac{1}{5} \qquad \text{(E)}\ 1\dfrac{10}{40}$

Solution

The fraction in question is equal to $1.25$. Expressing all of the choices we have,

$\text{(A)}\ 1.25 \qquad \text{(B)}\ 1.25 \qquad \text{(C)}\ 1.25 \qquad \text{(D)}\ 1.2 \qquad \text{(E)}\ 1.25$.

It's clear then that choice $\boxed{\text{D}}$ is the answer.

See Also

1992 AJHSME (ProblemsAnswer KeyResources)
Preceded by
Problem 1
Followed by
Problem 3
1 2 3 4 5 6 7 8 9 10 11 12 13 14 15 16 17 18 19 20 21 22 23 24 25
All AJHSME/AMC 8 Problems and Solutions

The problems on this page are copyrighted by the Mathematical Association of America's American Mathematics Competitions. AMC logo.png